5
$\begingroup$

Let X be a smooth projective variety over the complex numbers, of dimension at least two. $D$ is an ample divisor on X. Then we know for $m>>0$, $H^i(mD)=0$. Now suppose $E$ is another divisor that is algebraiclly equivalent to $mD$, i.e. the line bundle $\mathcal{L}(E-mD)\in Pic^0(X)$ lies in the identity component of of Picard variety of X. Then is it true that even if they are not linearly equivalent, we still have $dim H^0(\mathcal{L}(E))=dim H^0(\mathcal{L}(mD))$, for $m>>0$ ?

Since the Euler character is a topological invariant, we know $\chi(\mathcal{L}(E))=\chi(\mathcal{L}(mD))$. Therefore if we know $H^i(\mathcal{L}(E))=0$ for $i>0$, we are done. However it is not obvious to me if that is true or not.

Some of my mumbling which may or may not be related:

For a general line bundle $\mathcal{L}\in Pic^0(X)$, if $\mathcal{L}\neq \mathcal{O}_X$, then $H^0(\mathcal{L})=0$ since for effective divisor in a projective variety we have a notion of degree, see Hartchorne chap II Exer 6.2. But I don't know if $H^i(\mathcal{L})=0$ or not.

In a series of paper by Green and Lazarsfeld, they looked at the case where X is compact Kahler, not necessarily projective, the behavior where $\mathcal{L}\in Pic^0(X)$ but $H^i(\mathcal{L})\neq 0$. see paper. But I don't know how to use that to construct an example where $E\sim_{alg}mD$ but $H^i(\mathcal{L}(E))\neq 0$ for $i>0$, or $H^0(\mathcal{L}(E))\neq H^0(\mathcal{L}(mD))$.

$\endgroup$
1

1 Answer 1

5
$\begingroup$

What you want follows easily from the Kodaira vanishing theorem:

If $m$ is sufficiently large then $mD - K$, where $K$ is the canonical divisor, is ample (this is true for $K$ replaced by any divisor). Since ampleness is preserved by algebraic equivalence, for example by Kleiman's criterion, it follows that $E - K$ is also ample if $E$ is algebraically equivalent to $mD$. Kodaira's vanishing theorem then implies that $H^i(\mathcal{L}(E)) = 0$ for $i>0$.

$\endgroup$
1
  • 1
    $\begingroup$ Thanks, I realized this is a stupid question earlier today... $\endgroup$
    – Ying Zhang
    Sep 9, 2011 at 18:14

Your Answer

By clicking “Post Your Answer”, you agree to our terms of service and acknowledge you have read our privacy policy.

Not the answer you're looking for? Browse other questions tagged or ask your own question.